Difference between revisions of "2025 SSMO Speed Round Problems/Problem 9"
(Created page with "==Problem== Let <math>ABC</math> be a triangle. The point <math>P</math> lies on side <math>BC,</math> the point <math>Q</math> lies on side <math>AB,</math> and the point <m...") |
(→Solution) |
||
Line 4: | Line 4: | ||
==Solution== | ==Solution== | ||
+ | |||
+ | <asy> | ||
+ | unitsize(1.3cm); | ||
+ | |||
+ | pair A,B,C,P,Q,R,H,X,Y; | ||
+ | |||
+ | A=(1.75,3.5); | ||
+ | B=(0,0); | ||
+ | C=(8,0); | ||
+ | P=(3,0); | ||
+ | |||
+ | X=(1.5,0); | ||
+ | Y=(5.5,0); | ||
+ | |||
+ | Q=(1.5,3); | ||
+ | R=(5.5,1.4); | ||
+ | |||
+ | H=(1.75,0); | ||
+ | |||
+ | fill(A--Q--P--R--cycle,palegreen); | ||
+ | |||
+ | draw(A--B--C--cycle); | ||
+ | draw(Q--P--R); | ||
+ | draw(Q--X,red+dashed); | ||
+ | draw(R--Y,red+dashed); | ||
+ | draw(A--H,dashed); | ||
+ | |||
+ | label("$A$",A,dir(90)); | ||
+ | label("$B$",B,dir(225)); | ||
+ | label("$C$",C,dir(300)); | ||
+ | |||
+ | label("$X$",(1.4,0),dir(270)); | ||
+ | label("$Y$",Y,dir(270)); | ||
+ | |||
+ | label("$P$",P,dir(270)); | ||
+ | label("$Q$",(1.4,2.9),dir(120)); | ||
+ | label("$R$",(5.5,1.5),dir(90)); | ||
+ | |||
+ | label("$H$",(1.85,0),dir(270)); | ||
+ | |||
+ | draw(rightanglemark(A,H,P,4)); | ||
+ | draw(rightanglemark(Q,X,B,4)); | ||
+ | draw(rightanglemark(R,Y,P,4)); | ||
+ | |||
+ | dot(A); | ||
+ | dot(B); | ||
+ | dot(C); | ||
+ | dot(P); | ||
+ | dot(Q); | ||
+ | dot(R); | ||
+ | dot(X); | ||
+ | dot(Y); | ||
+ | dot(H); | ||
+ | </asy> | ||
+ | |||
+ | Let <math>X</math> and <math>Y</math> be the projections of <math>Q</math> and <math>R</math> onto <math>BC</math>, respectively. Since <math>\triangle BQP</math> and <math>\triangle CRP</math> are both isosceles, <math>X</math> and <math>Y</math> are the midpoints of <math>BP</math> and <math>CP</math>, respectively. Let <math>BH = x</math>, <math>CH = y</math>, and <math>AH = h</math>. Note that <math>x+y = BC = 8</math>, and also note that the condition <math>\angle APB < 90^\circ</math> implies <math>x=BH < BP = 3</math>. | ||
+ | |||
+ | Next, observe that <math>\triangle QBX \sim \triangle ABH</math>, and hence <math>\tfrac{QX}{BX} = \tfrac{AH}{BH}</math>. From, this, we get <math>QX = \tfrac{AH\cdot BX}{BH} = \tfrac{3h}{2x}</math>. Analogously, from the similarity <math>\triangle RCY \sim \triangle ACH</math>, we get <math>RY = \tfrac{5h}{2y}</math>. Thus,<cmath>[BQP] + [CRP] = \frac{9h}{4x} + \frac{25h}{4y}.</cmath>Now, using the area condition in the problem statement, we have | ||
+ | \begin{align*} | ||
+ | \frac{9h}{4x} + \frac{25h}{4y} &= [BQP] + [CRP] \\ | ||
+ | &= [ABC] - [AQPR] \\ | ||
+ | &= \tfrac{4}{7}[ABC] \\ | ||
+ | &= \frac{16h}{7}. | ||
+ | \end{align*}Cancelling a factor of <math>h</math> from the first and last equations in the above chain and substituting <math>y=8-x</math>, we obtain<cmath>\frac{9}{4x} + \frac{25}{4(8-x)} =\frac{16}{7}.</cmath>Solving this equation, we find <math>x = \tfrac{7}{4}, \tfrac{9}{2}</math>. Since <math>x<3</math>, we must have <math>x = \tfrac{7}{4}</math>, which implies <math>y =\tfrac{25}{7}</math>. Hence, <math>BH \cdot CH = xy = \tfrac{175}{16}</math>, and we extract <math>175+16 = \boxed{191}</math>. | ||
+ | |||
+ | ~Sedro |
Latest revision as of 22:45, 10 September 2025
Problem
Let be a triangle. The point
lies on side
the point
lies on side
and the point
lies on side
such that
and
. Let
be the foot of the altitude from
to
. Given that
and
the value of
can be expressed in the form
where
and
are relatively prime positive integers. Find
.
Solution
Let and
be the projections of
and
onto
, respectively. Since
and
are both isosceles,
and
are the midpoints of
and
, respectively. Let
,
, and
. Note that
, and also note that the condition
implies
.
Next, observe that , and hence
. From, this, we get
. Analogously, from the similarity
, we get
. Thus,
Now, using the area condition in the problem statement, we have
\begin{align*}
\frac{9h}{4x} + \frac{25h}{4y} &= [BQP] + [CRP] \\
&= [ABC] - [AQPR] \\
&= \tfrac{4}{7}[ABC] \\
&= \frac{16h}{7}.
\end{align*}Cancelling a factor of
from the first and last equations in the above chain and substituting
, we obtain
Solving this equation, we find
. Since
, we must have
, which implies
. Hence,
, and we extract
.
~Sedro